1 research outputs found

    Some Bounds for the Number of Blocks III

    Full text link
    Let D=(Ξ©,B)\mathcal D=(\Omega, \mathcal B) be a pair of vv point set Ξ©\Omega and a set B\mathcal B consists of kk point subsets of Ξ©\Omega which are called blocks. Let dd be the maximal cardinality of the intersections between the distinct two blocks in B\mathcal B. The triple (v,k,d)(v,k,d) is called the parameter of B\mathcal B. Let bb be the number of the blocks in B\mathcal B. It is shown that inequality (vd+2iβˆ’1)β‰₯b{(kd+2iβˆ’1)+(kd+2iβˆ’2)(vβˆ’k1)+....{v\choose d+2i-1}\geq b\{{k\choose d+2i-1} +{k\choose d+2i-2}{v-k\choose 1}+.... .+(kd+i)(vβˆ’kiβˆ’1)}.+{k\choose d+i}{v-k\choose i-1} \} holds for each ii satisfying 1≀i≀kβˆ’d1\leq i\leq k-d, in the paper: Some Bounds for the Number of Blocks, Europ. J. Combinatorics 22 (2001), 91--94, by R. Noda. If bb achieves the upper bound, D\mathcal D is called a Ξ²(i)\beta(i) design. In the paper, an upper bound and a lower bound, (d+2i)(kβˆ’d)i≀v≀(d+2(iβˆ’1))(kβˆ’d)iβˆ’1 \frac{(d+2i)(k-d)}{i}\leq v \leq \frac{(d+2(i-1))(k-d)}{i-1} , for vv of a Ξ²(i)\beta(i) design D\mathcal D are given. In the present paper we consider the cases when vv does not achieve the upper bound or lower bound given above, and get new more strict bounds for vv respectively. We apply this bound to the problem of the perfect ee-codes in the Johnson scheme, and improve the bound given by Roos in 1983.Comment: Stylistic corrections are made. References are adde
    corecore